Đến nội dung

apollo_1994 nội dung

Có 91 mục bởi apollo_1994 (Tìm giới hạn từ 05-06-2020)



Sắp theo                Sắp xếp  

#191813 Số vô tỉ

Đã gửi bởi apollo_1994 on 02-10-2008 - 14:27 trong Số học

Chứng minh rằng
a=0,123456789......( sau dấu phẩy là các số tự nhiên liên tiếp ) là một số vô tỉ

Bài này có ở đâu đó trên diễn đàn rùi
Sơ sơ là thế này:Giả sử số đó có dạng 0,a(b) với chu kì b gồm k chữ số
Xét số $10^{2k}$ có trong phần thập phân ấy=> phần thập phân gồm toàn các chữ số 0->vô lý



#191814 Đơn giản

Đã gửi bởi apollo_1994 on 02-10-2008 - 14:36 trong Các dạng toán khác

Tiện thể anh làm giùm em luôn bài này
Trong mặt phẳng cho 2000 điểm A_1,A_2,...,A_{2000}; chứng minh có đường thẳng không cắt điểm nào; chia mặt phẳng làm 2 miền mà mỗi miền chứa đúng 1000 điểm

Về lý thuyết thì điều này hoàn toàn có thể :D
Vẽ 1 đường tròn chứa trong đó 2000 điểm đã cho.
Nối đôi một các điểm ấy cắt đường tròn tạo thành các dây cung,lấy 1 điểm M bất kỳ thuộc đường tròn không trùng với đầu mút của 1 dây cung nào.
Từ điểm M kẻ các tia $MA_1,MA_2,..M{2000}$ theo chiều kim đồng hồ( đương nhiên là các tia này phân biệt)
Kẻ 1 tia Mx nằm giữa 2 tia $MA_{1000}$ và $MA_{1001}$,đường thẳng chứa tia đó là đt cần tìm->OK



#191830 Đơn giản

Đã gửi bởi apollo_1994 on 02-10-2008 - 21:57 trong Các dạng toán khác

Mình không hiểu bạn vẽ đường tròn để làm gì

Vẽ đường tròn là để tìm ra điểm M sao cho các tia $M_i$ không có tia nào trùng nhau

chỉ cần lấy 1 điểm M nào đó ngoài 2000 điểm đã cho sao cho có thể kẻ các tia M1; M2;...;M2000 theo chiều kim đòng hồ nối 2000 điểm đã cho là được

Cái này làm thế nào hả bạn :D,nếu trong bài mà bạn nói thế này thì người khác sẽ cho là mò đấy

Mà cách này thì có vẻ hơi mơ hồ

Không biết có phải mình hơi bảo thủ không,nhưng mình thấy mình trình bày thế là rõ ràng rồi chứ,mơ hồ ở chỗ nào nhỉ:D.
Như mình đã nói thì về lý thuyết thì có thể thực hiện được,còn trên thực tế thì....
Nếu bạn có cách khác hay hơn hì có thể post lên cho mọi người tham khảo ^^



#191848 xin các bạn và thày cô giải giúp!

Đã gửi bởi apollo_1994 on 03-10-2008 - 14:56 trong Hình học

1/ cho Tam giác ABC có góc B = 70 độ, góc C =50 độ. Đường cao AH, trung tuyến AM.
Tính góc HAM.

2/ Cho tam giác ABC cân tại A, góc A=20 độ, I là trung điểm của AC.
Tính góc IBC.

Mấy bài toán tính góc này mình học hồi lớp 7.Nói chung mấy bài này thì thường là người ta lấy 1 cạnh đã cho làm cạnh rồi dựng các tam giác vuông cân,tam giác đều,tam giác nửa đều... ,nối linh tinh lại,sau đó tìm các tam giác bằng nhau có liên quan đến góc cân tính....
Có thể tham khảo thêm trong sách "Nâng cao và phát triển toán 7" của Vũ Hữu Bình,trong đó có hẳn 1 chuyên đề về vấn đề này(nếu như mình nhớ không nhầm^^)



#191849 Nhìn thì dễ nhưng......

Đã gửi bởi apollo_1994 on 03-10-2008 - 15:17 trong Phương trình, hệ phương trình và bất phương trình

$\dfrac{1}{5 \sqrt[3]{3}-2 \sqrt[3]{9}+1 }= \dfrac{\sqrt[3]{3}}{(5\sqrt[3]{3}+6)(\sqrt[3]{3}-1)}$
Nhân liên hợp là ok



#192869 hình

Đã gửi bởi apollo_1994 on 26-10-2008 - 19:24 trong Hình học

I là tâm đường tròn nội tiếp tam giác ABC cạnh AB=c,AC=b,AB=c.CMR
$\dfrac{IA^2}{bc} + \dfrac{IB^2}{ca} + \dfrac{IC^2}{ab} =1$



#193565 lam thu xem sao.

Đã gửi bởi apollo_1994 on 18-11-2008 - 15:19 trong Bất đẳng thức và cực trị

lam thu bai nay xem sao.
*a,b,c>0 thoa3 a+b+c=3.Ch/m:
1)$\sqrt[5]{2a+b}+\sqrt[5]{2b+c}+\sqrt[5]{2c+a}=<3\sqrt[5]{3}$.


$\sqrt[5]{2a+b}.\sqrt[5]{81} \leq \dfrac{2a+b+12}{5} $
vv..



#194477 pt

Đã gửi bởi apollo_1994 on 06-12-2008 - 20:57 trong Phương trình, hệ phương trình và bất phương trình

<spam>Nếu bạn muốn lời giải gọn nhất thì hãy xem trong TTT tháng sau.Còn bài này chưa hết hạn gửi,bạn không nên post lên đây như vậy.



#197274 Đề thi chuyên lớp...2

Đã gửi bởi apollo_1994 on 07-05-2009 - 20:02 trong Các dạng toán khác

Hừm,cái chữ A đấy không có trong đề đâu,trong đề nó chỉ cho hình làm jè có kí hiệu :geq
Cũng không hiểu sao nghe đồn có đứa làm hết và được điểm tối đa mới hãi chứ :D



#198119 Bài min hay đây

Đã gửi bởi apollo_1994 on 19-05-2009 - 18:18 trong Bất đẳng thức và cực trị

Cho x,y là hai số dương thỏa mãn
$ x+\dfrac{1}{y} \leq 1$
Tìm giá trị min nhỏ nhất của biểu thức $M=32 \dfrac{x}{y} +1999 \dfrac{y}{x} $

$M=(32.\dfrac{x}{y}+ 2\dfrac{y}{x})+1997.\dfrac{x}{y}$
$\geq \sqrt{32.2} +1997.\dfrac{4}{(x+\dfrac{1}{y})^2 } \geq 8004$
Đẳng thức khi $x=0,5;y=2$



#198226 Hệ khó xơi

Đã gửi bởi apollo_1994 on 20-05-2009 - 19:44 trong Phương trình, hệ phương trình và bất phương trình

Tìm nghiệm nguyên của hê phương trình:
$x + y + z = 3(1)$
$x^3 + y^3 + z^3 = 3$

$(1) \Leftrightarrow x^3+y^3+z^3+3(x+y)(y+z)(z+x)=27$
$\Leftrightarrow (x+y)(y+z)(z+x)=8$
-->xét ước...
---------
Đây bạn có thể xem thêm:
ta tìm cách phân tích 8 thành 3 thừa số có tổng bằng 6 vì $(x+y)+(y+z)+(z+x)=6$
chỉ có 2 cách phân tích là $8=2.2.2=(-1).(-1).8$
Đến đây chắc bạn tự lam đc rồi chứ?



#198580 Phần nguyên

Đã gửi bởi apollo_1994 on 24-05-2009 - 21:25 trong Kinh nghiệm học toán

Mấy anh ơi, em lớp 9, mún bik nhiều về phần nguyên và phương pháp, em nghe nói TTT có chuyên đề jì về phần nguyên hay lắm nhưng em ko đặt báo, anh nào có kinh nghiệm truyền thụ cho em nha :))Em sắp thi òi :D
có ebooks càng tốt ạ :D

Bạn tìm trong quyển số của bộ sách ĐHSP ý,có hẳn 1 chuyên đề về cái này.Trong TTT cũng chỉ có 1 số VD xoay kĩ về 1 dạng bài trong đó thui.
Nói thật là vô cùng đau sốt với cái thể loại phần nguyên này :Leftrightarrow



#198984 toán cực trị!

Đã gửi bởi apollo_1994 on 27-05-2009 - 09:57 trong Bất đẳng thức và cực trị

Nhờ các bạn giải giùm với!!!
Tìm Max P= (a-b)4+ (b-c)4+ (c-a)4
Cho a,b,c là các số thực không nhỏ hơn 1 và không lớn hơn 2.

Từ giả thiết suy ra $0 \leq |a-b|,|b-c|,|c-a| \leq 1$
Do đó $P \leq |a-b|+|b-c|+|c-a|$
Không mất tổng quát giả sử $a \geq b,a \geq c$
Khi đó $P \leq 2a-b-c-|b-c|$
+Nếu $b \geq c$ thì $P \leq 2(a-b) \leq 2(2-1)=1$
+Nếu $b \leq c$ tương tự
Vậy $maxP=2$ khi có 2 số bằng 2,1 số bằng 1



#199508 Bất đẳng thức

Đã gửi bởi apollo_1994 on 30-05-2009 - 22:43 trong Bất đẳng thức - Cực trị

Cho a,b,c > 0 và$ a^{2}+b^{2}+c^{2}=3$.Tìm GTNN:$ A= \dfrac{5}{3}(a+b+c)+\dfrac{1}{a}+\dfrac{1}{b}+\dfrac{1}{c}$

Dễ thấy $a,b,c \leq 3$
Vì vậy $(a-1)^2.(a-3) \leq 0$
$\Leftrightarrow a^3-5a^2+7a-3 \leq 0$
$\Leftrightarrow \dfrac{5}{3}a+ \dfrac{1}{a} \geq \dfrac{a^2}{3} + \dfrac{7}{3} $
Tương tự: $\dfrac{5}{3}b+ \dfrac{1}{b} \geq \dfrac{b^2}{3} + \dfrac{7}{3} $
$\dfrac{5}{3}c+ \dfrac{1}{c} \geq \dfrac{c^2}{3} + \dfrac{7}{3}$
Cộng theo vế có $A \geq 8$
Vậy là OK!



#199682 giải giúp zoi pa con

Đã gửi bởi apollo_1994 on 01-06-2009 - 11:18 trong Phương trình, hệ phương trình và bất phương trình

tìm nghiệm nguyên của phương trình:
$x(x^2+x+1)=4y(y+1)$

$ \Leftrightarrow (x+1)(x^2+1)=(2y+1)^2$
$ \Rightarrow x+1,x^2+1$ lẻ $ \Rightarrow (x+1;x^2+1)=1$(dễ thấy :P)
Nên cả 2 đều là số chính phương........:P



#200047 Bất đẳng thức

Đã gửi bởi apollo_1994 on 03-06-2009 - 18:03 trong Bất đẳng thức - Cực trị

a,tìm x, y thỏa mãn PT:$ (x^{2}-y^{2}+2)^{2}+4x^{2}y^{2}+6x^{2}-y^{2}=0$ sao cho$ P=x^{2}+y^{2}$ đạt GTNN

b,bài này mọi người làm nhiều cách nhé:
CMr: với $x,y,z>0$ thì $\dfrac{2x}{x^{6}+y^{4}}+ \dfrac{2y}{y^{6}+z^{4}}+ \dfrac{2z}{z^{6}+x^{4}}\leq \dfrac{1}{x^{4}}+\dfrac{1}{y^{4}}+\dfrac{1}{z^{4}}$

a)$y^2=P-x^2$
Thay vào pt có:$15x^2+p^2-5p+4=0$
$\Leftrightarrow (p-1)(p-4)=-15x^2 \leq 0 \Rightarrow P \geq 1 \Leftrightarrow x=0,y= \pm 1$
b) $\sum \dfrac{2x}{x^{6}+y^{4}} \leq \sum \dfrac{2x}{2x^3y^2}= \sum \dfrac{1}{x^2y^2} \leq \sum \dfrac{1}{x^4} $
OK!



#200099 Giải phương trình

Đã gửi bởi apollo_1994 on 04-06-2009 - 07:48 trong Phương trình, hệ phương trình và bất phương trình

ta sẽ chọn a;b sao cho thỏa mãn hai phương trình 1 và 2 của hệ có hệ số giống nhau,tức là:
$\dfrac{4}{{{a^2}}} = \dfrac{{ - 12}}{{2ab}} = \dfrac{{ - a}}{{ - 2}} = \dfrac{{5 - b}}{{{b^2} - 1}}$


Chỗ này nhầm chút xíu mà :D.Vẫn ra được $a=2,b=-3$ đấy thui:D
Rất cám ơn các bác :D.Đúng là cái em đang cần ạ.



#200141 hình tứ diện

Đã gửi bởi apollo_1994 on 04-06-2009 - 16:02 trong Hình học

bài 2:
cho ABCD là hình bình hành,E,F thuộc AB sao cho E gần A hơn.CE cắt DF tại P. Q là giao điểm thứ 2 của 2 đường tròn nội tiếp PAE vàPBF.CM PQ //AD

$PQ \cap AB=I$
$PQ \cap CD=K$
Theo phương tích:$IE.IA=IF.IB(=IP.IQ) \Rightarrow \dfrac{IA}{IB}=\dfrac{IF}{IA}$
Mặt khác $\dfrac{IF}{IE}=\dfrac{KD}{KC}$(dễ thấy)
Do đó $\dfrac{IA}{IB}=\dfrac{KD}{KC} \Rightarrow IK//AD$ hay $PQ//AD $



#200144 Mệnh đề tương đương

Đã gửi bởi apollo_1994 on 04-06-2009 - 16:15 trong Đại số

Bài 1, Cho 2 sô không âm x,y thỏa mãn $x^{3} + y^{3} =2 $
CMR : $x^2 + y^2 \leq 2 $

Bài 2, Cho a,b thỏa mãn $a+b \geq 0 $
CMR: với $n \in N$ thì $(\dfrac{a+b}{2})^{n} \leq \dfrac{a^{n}+b^{n}}{2} $

Bài 1:
Cô-si:
$x^3+x^3+1 \geq 3x^2$
$y^3+y^3+1 \geq 3y^2$
suy ra $2(x^3+y^3)+2 \geq 3(x^2+y^2) \Rightarrow x^2+y^2 \leq 2$
Bài 2
Bạn dùng quy nạp và áp dụng $(a+b)(a^n+b^n) \geq 2(a^{n+1}+b^{n+1})$



#200145 Phương trình nghiệm nguyên, số chính phương

Đã gửi bởi apollo_1994 on 04-06-2009 - 16:36 trong Số học

giải PT nghiệm nguyên : $x^{2}+y^{2}+z^{2}=x^{2}y^{2}$

Ông anh này tự sướng à :D
Bài trên thì cóa thể làm như vầy:
+Nếu $x^2y^2$ lẻ thì $z^2$ lẻ
=> VT chia 4 dư 3,Vp chia 4 dư 1=>vô no
+Nếu $x^2y^2$ chẵn ,giả sử x chẵn thì $y^2+z^2 \vdots 4 \Rightarrow y \vdots 2,z \vdots 2$
Tới đây xuống thang,pt có nghiệm duy nhất $(0;0;0)$



#200146 Bất đẳng thức

Đã gửi bởi apollo_1994 on 04-06-2009 - 16:44 trong Bất đẳng thức - Cực trị

Cho ké phát :D
Cho $a,b,c>0$ tìm min:
$P= \dfrac{b(a-c)}{c(a+b)} + \dfrac{c(3b+a)}{a(b+c)} + \dfrac{3c(a-b)}{b(a+c)} $
[trích đề thi thử KHTN]
Đi thi kiểu này chắc rớt quá T_T



#200318 Phương trình nghiệm nguyên, số chính phương

Đã gửi bởi apollo_1994 on 05-06-2009 - 21:53 trong Số học

cho n là số tự nhiên .Tìm n sao cho :$n^{2}+n+2 \vdots 2005$

$n^{2}+n+2 \not \vdots 5 \forall n \in N$ nên $\not \exists n$
Còn bài bên trên là đề thi KHTN năm nào đấy :),xét số dư của 1 số CP cho 17 là OK!



#200323 BĐT hình học

Đã gửi bởi apollo_1994 on 05-06-2009 - 22:07 trong Hình học

cho điểm O bên trong ta ggiác ABC sao cho tia AO,BO,CO cắt BC,AC,AB tại D,E,F . Tìm GTNN của:$M=\dfrac{OA}{OD}.\dfrac{OB}{OE}.\dfrac{OC}{OF}$

Đặt $S_{AOB}=S_1,S_{AOC}=S_2,S_{BOC}=S_3$
$M= \dfrac{S_1+S_2}{S_3}.\dfrac{S_2+S_3}{S_2}.\dfrac{S_3+S_1}{S_2} \geq 8$



#200329 BĐT hình học

Đã gửi bởi apollo_1994 on 05-06-2009 - 22:16 trong Hình học

Làm bài này nhiều cách : cho tam giác ABC nhọn và M nằm trong tam giác .Tìm GTNN của: $Q=MA.BC+MB.AC+MC.AB$

Hạ $AH,MK \perp BC$
Ta có $AM+MK \geq AH \Rightarrow (AM+MK).BC \geq AH.BC$ hay $AM.BC \geq 2(S_{ABC}-S_{BMC})$
Làm 2 cái tương tự và cộng lại : $Q \geq 4S$ dấu = khi M là trực tâm.
Tạm thế đã,chưa nghĩ ra cách khác :).Ngu lâu khó đào tạo :)



#200531 hayhyhay

Đã gửi bởi apollo_1994 on 07-06-2009 - 08:05 trong Số học

Cho a,b là 2 số nguyên dương thoả mãn:
$a^{1954}+b^{1954}, a^{1945}+b^{1945}$ chia hết cho 2001.Hỏi $a^{2002}+b^{2002}$ có chia hết cho 2001 không?
(sài Ferma)

$(a^{977})^2+(b^{977})^2 \vdots 3 \Rightarrow \left\{ a^{977} \vdots 3 \\ b^{977} \vdots 3 \\ $
$\Rightarrow \left\{a \vdots 3 \\ b \vdots 3 \\ $
$(a^{977})^2+(b^{977})^2 \vdots 667 \Rightarrow \left\{ a^{977} \vdots 667 \\ b^{977} \vdots 667 \\ $
$\Rightarrow \left\{a \vdots 667 \\ b \vdots 667 \\ $
Suy ra $a^{2002}+b^{2002} \vdots 2001$

Vậy là cái 1945 kia là thừa à :) Không lẽ mình làm sai?